Diễn Đàn MathScopeDiễn Đàn MathScope
  Diễn Đàn MathScope
Ghi Danh Hỏi/Ðáp Thành Viên Social Groups Lịch Ðánh Dấu Ðã Ðọc

Go Back   Diễn Đàn MathScope > Sơ Cấp > Đại Số và Lượng Giác

News & Announcements

Ngoài một số quy định đã được nêu trong phần Quy định của Ghi Danh , mọi người tranh thủ bỏ ra 5 phút để đọc thêm một số Quy định sau để khỏi bị treo nick ở MathScope nhé !

* Nội quy MathScope.Org

* Một số quy định chung !

* Quy định về việc viết bài trong diễn đàn MathScope

* Nếu bạn muốn gia nhập đội ngũ BQT thì vui lòng tham gia tại đây

* Những câu hỏi thường gặp

* Về việc viết bài trong Box Đại học và Sau đại học


Trả lời Gởi Ðề Tài Mới
 
Ðiều Chỉnh Xếp Bài
Old 17-07-2012, 12:43 PM   #1
Snow Bell
+Thành Viên Danh Dự+
 
Tham gia ngày: Apr 2012
Đến từ: Heaven
Bài gởi: 579
Thanks: 10
Thanked 513 Times in 283 Posts
Topic về Bất Đẳng Thức (3)

Chào các bạn,

Như đã thấy, Topic về Bất Đẳng Thức cũ đã khá nhiều nên mình cắt thành topic thứ 3. Hy vọng các bạn sẽ tuân thủ đúng nội quy nêu ra sau đây:

1. Đánh số thứ tự bài:

Người post bài mới phải đánh số thứ tự. Bài đầu tiên sẽ được đánh số 1, và cứ thế các bài đề nghị tiếp theo sẽ được đánh số kế tiếp.

-------------------------------------------------------
2. Không post chen ngang:

KHÔNG có những comments rác chen ngan.
Post lời giải bài cũ trước khi post đề mới.

--------------------------------------------------------

3. Và tất nhiên, mỗi post đều phải được đánh Latex rõ ràng.

Những bài viết không đạt yêu cầu, đều bị xóa không báo trước.

Mong mọi người hãy tuân thủ để tạo nên Topic Bất Đẳng Thức này đẹp và hấp dẫn hơn.

Xin cảm ơn.
[RIGHT][I][B]Nguồn: MathScope.ORG[/B][/I][/RIGHT]
 

thay đổi nội dung bởi: ptk_1411, 22-07-2012 lúc 07:57 AM
Snow Bell is offline   Trả Lời Với Trích Dẫn
The Following 13 Users Say Thank You to Snow Bell For This Useful Post:
Akira Vinh HD (21-07-2012), batigoal (17-07-2012), hgly1996 (26-07-2012), hoduckhanhgx (22-07-2012), hunterkill17 (17-07-2012), JokerNVT (17-07-2012), Ng_Anh_Hoang (18-07-2012), sang89 (17-07-2012), thephuong (17-07-2012), tienanh_tx (27-08-2012), TrauBo (17-07-2012), Trieu123 (05-02-2015), zớt (13-08-2012)
Old 17-07-2012, 01:22 PM   #2
Snow Bell
+Thành Viên Danh Dự+
 
Tham gia ngày: Apr 2012
Đến từ: Heaven
Bài gởi: 579
Thanks: 10
Thanked 513 Times in 283 Posts
Mình mở đầu bằng bài toán sau.
Bài 1: Cho $ a,b,c \ge 0 $ thỏa $ a+b+c+abc=4 $.Chứng minh rằng:
$$ ab+bc+ca \le a+b+c $$
[RIGHT][I][B]Nguồn: MathScope.ORG[/B][/I][/RIGHT]
 
Snow Bell is offline   Trả Lời Với Trích Dẫn
The Following 4 Users Say Thank You to Snow Bell For This Useful Post:
Akira Vinh HD (15-07-2016), High high (25-07-2012), phamvanhuy (01-11-2014), Trieu123 (05-02-2015)
Old 17-07-2012, 01:53 PM   #3
vjpd3pz41iuai
+Thành Viên+
 
vjpd3pz41iuai's Avatar
 
Tham gia ngày: Dec 2011
Bài gởi: 303
Thanks: 129
Thanked 130 Times in 81 Posts
Trích:
Nguyên văn bởi Vinh Phuc View Post
Mình mở đầu bằng bài toán sau.
Bài 1: Cho $ a,b,c \ge 0 $ thỏa $ a+b+c+abc=4 $.Chứng minh rằng:
$$ ab+bc+ca \le a+b+c $$
Từ điều kiện bài toán thì tồn tại 3 số $x,y,z$ sao cho
$a=\dfrac{2x}{y+z},b=\dfrac{2y}{x+z},c=\dfrac{2z}{ x+y}$
Thay vào cái cần chứng minh phá tung ra quy đồng nhân chéo ta được bđt
$a^{3}+b^{3}+c^{3}+3abc\geq ab(a+b)+bc(b+c)+ca(c+a)$
BĐT này là Schur quen thuộc.
[RIGHT][I][B]Nguồn: MathScope.ORG[/B][/I][/RIGHT]
 
__________________

thay đổi nội dung bởi: Trầm, 19-07-2012 lúc 12:03 PM
vjpd3pz41iuai is offline   Trả Lời Với Trích Dẫn
The Following 2 Users Say Thank You to vjpd3pz41iuai For This Useful Post:
caominh (19-09-2014), greg_51 (13-06-2014)
Old 17-07-2012, 02:17 PM   #4
Trầm
+Thành Viên Danh Dự+
 
Tham gia ngày: Feb 2011
Bài gởi: 657
Thanks: 388
Thanked 470 Times in 196 Posts
Bài 2:
Cho các số dương $a, b, c$ thỏa mãn $abc=1$. Chứng minh rằng:
$$\dfrac{1}{a^2-a+1}+\dfrac{1}{b^2-b+1}+\dfrac{1}{c^2-c+1} \le 3$$
[RIGHT][I][B]Nguồn: MathScope.ORG[/B][/I][/RIGHT]
 
__________________

thay đổi nội dung bởi: Trầm, 17-07-2012 lúc 02:27 PM
Trầm is offline   Trả Lời Với Trích Dẫn
The Following 2 Users Say Thank You to Trầm For This Useful Post:
hanhphuc254 (15-09-2012), VYKA (05-08-2012)
Old 17-07-2012, 02:25 PM   #5
vjpd3pz41iuai
+Thành Viên+
 
vjpd3pz41iuai's Avatar
 
Tham gia ngày: Dec 2011
Bài gởi: 303
Thanks: 129
Thanked 130 Times in 81 Posts
Trích:
Nguyên văn bởi tanggo View Post
Bài 2:
Cho các số dương $a, b, c$ thỏa mãn $abc=1$. Chứng minh rằng:
$$\dfrac{1}{a^2-a+1}+\dfrac{1}{b^2-b+1}+\dfrac{1}{c^2-c+1} \le 3$$
Bđt cần CM tương đương với
$\frac{4}{3}-\frac{1}{a^{2}-a+1}+\frac{4}{3}-\frac{1}{b^{2}-b+1}+\frac{4}{3}-\frac{1}{c^{2}-c+1}\geq 1 $
$\Leftrightarrow \sum \frac{(2a-1)^{2}}{3(a^{2}-a+1)}\geq 1 $
Theo CS
$\sum \frac{(2a-1)^{2}}{3(a^{2}-a+1)}\geq \frac{[2(a+b+c)-3)]^{2}}{3[a^{2}+b^{2}+c^{2}-(a+b+c)+3]} $ $\geq 1 $
Nên cần CM $(a+b+c)^{2}+6(ab+bc+ac)\geq 9(a+b+c) $
Tới đây chắc đơn giản
[RIGHT][I][B]Nguồn: MathScope.ORG[/B][/I][/RIGHT]
 
__________________

thay đổi nội dung bởi: Trầm, 17-07-2012 lúc 02:59 PM
vjpd3pz41iuai is offline   Trả Lời Với Trích Dẫn
The Following User Says Thank You to vjpd3pz41iuai For This Useful Post:
High high (25-07-2012)
Old 17-07-2012, 02:50 PM   #6
MK.Duy
+Thành Viên+
 
MK.Duy's Avatar
 
Tham gia ngày: Dec 2011
Bài gởi: 33
Thanks: 100
Thanked 12 Times in 10 Posts
Bài 3
Cho $a,b,c,d >0, \frac{1}{a}+ \frac{1}{b}+ \frac{1}{c}+ \frac{1}{d}=4 $
Chứng minh rằng:
$\sqrt[3]{\frac{a^3+b^3}{2}}+\sqrt[3]{\frac{b^3+c^3}{2}}+\sqrt[3]{\frac{c^3+d^3}{2}}+\sqrt[3]{\frac{d^3+a^3}{2}} \leq 2(a+b+c+d)-4 $

[RIGHT][I][B]Nguồn: MathScope.ORG[/B][/I][/RIGHT]
 
__________________
http://violet.vn/11b1tohieu/entry/show/entry_id/4130539

thay đổi nội dung bởi: Trầm, 18-07-2012 lúc 11:31 AM
MK.Duy is offline   Trả Lời Với Trích Dẫn
The Following User Says Thank You to MK.Duy For This Useful Post:
High high (25-07-2012)
Old 18-07-2012, 12:41 AM   #7
keodua123
+Thành Viên+
 
Tham gia ngày: Mar 2011
Bài gởi: 111
Thanks: 74
Thanked 27 Times in 19 Posts
Trích:
Nguyên văn bởi MK.Duy View Post
Bài 3
Cho $a,b,c,d >0, \frac{1}{a}+ \frac{1}{b}+ \frac{1}{c}+ \frac{1}{d}=4 $
Chứng minh rằng:
$\sqrt[3]{\frac{a^3+b^3}{2}}+\sqrt[3]{\frac{b^3+c^3}{2}}+\sqrt[3]{\frac{c^3+d^3}{2}}+\sqrt[3]{\frac{d^3+a^3}{2}} \leq 2(a+b+c+d)-4 $
Trước hết ta chứng minh bất đẳng thức
\[\sqrt[3]{{\frac{{{a^3} + {b^3}}}{2}}} \le \frac{{{a^2} + {b^2}}}{{a + b}} \Leftrightarrow {\left( {a - b} \right)^4}\left( {{a^2} + ab + {b^2}} \right) \ge 0 \mbox{ (hiển nhiên đúng)}.\]
Suy ra
\[\sum {\sqrt[3]{{\frac{{{a^3} + {b^3}}}{2}}}} \le \sum {\frac{{{a^2} + {b^2}}}{{a + b}}}. \]
Ta có
\[2\sum {a} - \sum {\frac{{{a^2} + {b^2}}}{{a + b}} = 2\sum {\frac{1}{{\frac{1}{a} + \frac{1}{b}}}} } \ge 2.\frac{{{4^2}}}{{\sum {\frac{1}{a}} }} = 4.\]
Do đó
\[\sum {\sqrt[3]{{\frac{{{a^3} + {b^3}}}{2}}}} \le \sum {\frac{{{a^2} + {b^2}}}{{a + b}}} \le 2\sum a - 4. \mbox{ (điều phải chứng minh)}\]
Bài 4 (Vô địch Mĩ 2011).
Với $a, b, c$ là các số thực dương thỏa ${a^2} + {b^2} + {c^2} + {\left( {a + b + c} \right)^2} \le 4$, chứng minh rằng
\[\frac{{ab + 1}}{{{{\left( {a + b} \right)}^2}}} + \frac{{bc + 1}}{{{{\left( {b + c} \right)}^2}}} + \frac{{ca + 1}}{{{{\left( {c + a} \right)}^2}}} \ge 3.\]
[RIGHT][I][B]Nguồn: MathScope.ORG[/B][/I][/RIGHT]
 

thay đổi nội dung bởi: Trầm, 18-07-2012 lúc 11:32 AM
keodua123 is offline   Trả Lời Với Trích Dẫn
The Following User Says Thank You to keodua123 For This Useful Post:
High high (25-07-2012)
Old 18-07-2012, 02:43 AM   #8
tranghieu95
+Thành Viên+
 
tranghieu95's Avatar
 
Tham gia ngày: Oct 2010
Đến từ: THPT Phan Bội Châu- Nghệ An
Bài gởi: 382
Thanks: 187
Thanked 364 Times in 197 Posts
Gửi tin nhắn qua Yahoo chát tới tranghieu95
Trích:
Nguyên văn bởi keodua123 View Post
Bài 4 (Vô địch Mĩ 2011).
Với $a, b, c$ là các số thực dương thỏa ${a^2} + {b^2} + {c^2} + {\left( {a + b + c} \right)^2} \le 4$, chứng minh rằng
\[\frac{{ab + 1}}{{{{\left( {a + b} \right)}^2}}} + \frac{{bc + 1}}{{{{\left( {b + c} \right)}^2}}} + \frac{{ca + 1}}{{{{\left( {c + a} \right)}^2}}} \ge 3.\]
Ta có:
$\dfrac{ab+1}{(a+b)^2} \geq \dfrac{ab+\dfrac{a^2+b^2+c^2+ab+bc+ca}{2}}{(a+b)^2 }=\dfrac{(a+b)^2+(a+c)(b+c)}{2(a+b)^1}=\dfrac{1}{2 }+\dfrac{(a+c)(b+c)}{2(a+b)^2}$
Tương tự với $\dfrac{{bc + 1}}{{{{\left( {b + c} \right)}^2}}} $ và
$\dfrac{{ca + 1}}{{{{\left( {c + a} \right)}^2}}}$
$\Rightarrow VT\geq \dfrac{3}{2}+\dfrac{1}{2}.(\dfrac{(a+b)(a+c)}{(b+c )^2}+\dfrac{(b+c)(b+a)}{(a+c)^2}+\dfrac{(c+a)(c+b) }{(a+b)^2}\geq 3$
Dấu "=" $\Leftrightarrow a=b=c=1$
[RIGHT][I][B]Nguồn: MathScope.ORG[/B][/I][/RIGHT]
 
__________________
TỪ TỪ LÀ HẠNH PHÚC
A1K39
XIN LỖI ĐÃ THẤT HỨA NHÉ

KỆ
tranghieu95 is offline   Trả Lời Với Trích Dẫn
The Following 2 Users Say Thank You to tranghieu95 For This Useful Post:
High high (25-07-2012), Nvthe_cht. (19-06-2014)
Old 18-07-2012, 02:44 AM   #9
Nts_pbc
+Thành Viên+
 
Tham gia ngày: Aug 2011
Đến từ: ..Yên Thành, Nghệ̣ An..BoxMath.vn..
Bài gởi: 28
Thanks: 11
Thanked 24 Times in 14 Posts
Trích:
Nguyên văn bởi keodua123 View Post
Bài 4 (Vô địch Mĩ 2011).
Với $a, b, c$ là các số thực dương thỏa ${a^2} + {b^2} + {c^2} + {\left( {a + b + c} \right)^2} \le 4$, chứng minh rằng
\[\frac{{ab + 1}}{{{{\left( {a + b} \right)}^2}}} + \frac{{bc + 1}}{{{{\left( {b + c} \right)}^2}}} + \frac{{ca + 1}}{{{{\left( {c + a} \right)}^2}}} \ge 3.\]
Từ giả thiết ta suy ra $$a^2+b^2+c^2+ab+bc+ca\le 2$$
Do đó ta đi chứng minh BĐT mạnh hơn là
$$\frac{2ab+\sum a^2+\sum ab}{(a + b)^2}+\frac{2bc+\sum a^2+\sum ab}{(b+c)^2}+\frac{2ca+\sum a^2+\sum ab}{(c+a)^2}\ge 6$$
$$\Leftrightarrow \sum \frac{(a+b)^2+(c+a)(c+b)}{(a + b)^2}\ge 6$$
$$\Leftrightarrow \frac{(c+a)(c+b)}{(a+b)^2}+\frac{(a+b)(a+c)}{(b+c) ^2}+\frac{(b+c)(b+a)}{(c+a)^2}\ge 3$$
Theo BĐT $AM-GM$ ta dễ có điều này!

Mình xin góp vui 1 bài:
Bài 5: Cho $a,b,c$ là các số không âm và $a+b+c=1.$ Chứng minh rằng
$$\frac{1}{2-3ab}+\frac{1}{2-3bc}+\frac{1}{2-3ca}\le \frac{9}{5}$$
[RIGHT][I][B]Nguồn: MathScope.ORG[/B][/I][/RIGHT]
 
Nts_pbc is offline   Trả Lời Với Trích Dẫn
The Following User Says Thank You to Nts_pbc For This Useful Post:
High high (25-07-2012)
Old 18-07-2012, 08:30 PM   #10
zớt
+Thành Viên+
 
Tham gia ngày: Jul 2012
Đến từ: THPT Chuyên Phan Bội Châu - Nghệ AN
Bài gởi: 136
Thanks: 120
Thanked 45 Times in 28 Posts
Trích:
Nguyên văn bởi Nts_pbc View Post
Mình xin góp vui 1 bài:
Bài 5: Cho $a,b,c$ là các số không âm và $a+b+c=1.$ Chứng minh rằng
$\frac{1}{2-3ab}+\frac{1}{2-3bc}+\frac{1}{2-3ca}\le \frac{9}{5} $
$A=\frac{1}{2-3ab}+\frac{1}{2-3bc}+\frac{1}{2-3ac} = \frac{\sum (2-3bc)(2-3ac)}{(2-3ab)(2-3bc)(2-3ac)} $
Ta thấy bất đẳng thức trên nằm trong dạng $f(abc,ab+bc+ca,a+b+c) $.

Suy ra max của hàm trên đạt được khi có 2 số bằng nhau, giả sử $a=b=x. $
$A=\frac{1}{2-3x^2}+\frac{2}{2-3xc} $
Cùng với điều kiện $2x+c=1 \Leftrightarrow c=1-2x $, thế vào $A $ ta được:
$A=\frac{1}{2-3x^2}+\frac{2}{2-3x(1-2x)} $
$A'=0 \Leftrightarrow x=\frac{1}{3} $
Với $x=\frac{1}{3} $thì $A''<0. $
Suy ra $A $ đạt cực đại tại $x= \frac{1}{3} $ hay $a=b=c=\frac{1}{3} $ và $\max A =\frac{9}{5} $.
[RIGHT][I][B]Nguồn: MathScope.ORG[/B][/I][/RIGHT]
 

thay đổi nội dung bởi: magician_14312, 18-07-2012 lúc 09:17 PM
zớt is offline   Trả Lời Với Trích Dẫn
The Following 2 Users Say Thank You to zớt For This Useful Post:
greg_51 (13-06-2014), keodua123 (18-07-2012)
Old 18-07-2012, 08:56 PM   #11
zớt
+Thành Viên+
 
Tham gia ngày: Jul 2012
Đến từ: THPT Chuyên Phan Bội Châu - Nghệ AN
Bài gởi: 136
Thanks: 120
Thanked 45 Times in 28 Posts
Trích:
Nguyên văn bởi kedaumat View Post
Bạn chi tiết hơn chỗ này giúp mình nhé
Đây chính là phương pháp đánh giá tích bạn down về tham khảo
[Only registered and activated users can see links. ]

Bài 6 : Cho $a,b \in \mathbb{R} $. Tìm Max :
$ \frac{a+b}{(a^4+4)(b^4+4)} $

[RIGHT][I][B]Nguồn: MathScope.ORG[/B][/I][/RIGHT]
 
__________________

thay đổi nội dung bởi: ptk_1411, 22-07-2012 lúc 08:10 AM Lý do: Tự động gộp bài
zớt is offline   Trả Lời Với Trích Dẫn
The Following 6 Users Say Thank You to zớt For This Useful Post:
caominh (19-09-2014), JokerNVT (18-07-2012), NguyenThanhThi (18-07-2012), q785412369 (23-10-2012), thaygiaocht (31-08-2012), yourenotalone (08-08-2014)
Old 19-07-2012, 12:52 PM   #12
magician_14312
Moderator
 
magician_14312's Avatar
 
Tham gia ngày: Jan 2011
Đến từ: Solar System
Bài gởi: 367
Thanks: 201
Thanked 451 Times in 220 Posts
Trích:
Nguyên văn bởi Vinh Phuc View Post
Mình mở đầu bằng bài toán sau.
Bài 1: Cho $ a,b,c \ge 0 $ thỏa $ a+b+c+abc=4 $.Chứng minh rằng:
$$ ab+bc+ca \le a+b+c $$
Từ giả thiết $a+b+c+abc=4$ hay $p+r=4$ suy ra $3 \le p \le 4$. Ta cần chứng minh $ q \le p.$

Theo bất đẳng thức Schur thì $p^3+9r \ge 4pq$. Ta sẽ chứng minh $4p^2 \ge p^3+9r$.

Thật vậy, thay $r=4-p$ vào ta thu được bất đẳng thức luôn đúng:
$$4p^2 \ge p^3 +9(4-p)\Leftrightarrow (p-3)(p+3)(4-p) \ge 0.$$
Đẳng thức xảy ra khi $a=b=c=1$ hoặc $a=0, b=c=2$ và các hoán vị.
[RIGHT][I][B]Nguồn: MathScope.ORG[/B][/I][/RIGHT]
 
__________________
...THE MILKY WAY...

thay đổi nội dung bởi: Trầm, 19-07-2012 lúc 02:07 PM Lý do: Đã có rồi
magician_14312 is offline   Trả Lời Với Trích Dẫn
The Following 7 Users Say Thank You to magician_14312 For This Useful Post:
greg_51 (13-06-2014), hgly1996 (26-07-2012), keodua123 (19-07-2012), Trầm (19-07-2012), Trieu123 (06-02-2015), Trung_Tr.Anh (25-08-2013), vjpd3pz41iuai (19-07-2012)
Old 19-07-2012, 05:00 PM   #13
pexea12
+Thành Viên+
 
Tham gia ngày: Jul 2010
Đến từ: HUS
Bài gởi: 81
Thanks: 58
Thanked 56 Times in 35 Posts
Bài 7 Cho $a,b,c>0$ và $a+b+c=3$. Chứng minh rằng
$$\dfrac{1}{1+ab}+\dfrac{1}{1+bc}+\dfrac{1}{1+ca} \ge \dfrac{9}{2(\sqrt{a}+\sqrt{b}+\sqrt{c})}$$
[RIGHT][I][B]Nguồn: MathScope.ORG[/B][/I][/RIGHT]
 
__________________
"I don't quit once I step on court"

thay đổi nội dung bởi: pexea12, 19-07-2012 lúc 05:02 PM
pexea12 is offline   Trả Lời Với Trích Dẫn
The Following User Says Thank You to pexea12 For This Useful Post:
caominh (19-09-2014)
Old 19-07-2012, 06:03 PM   #14
Ispectorgadget
+Thành Viên+
 
Tham gia ngày: Dec 2011
Đến từ: Hồ Chí Minh city
Bài gởi: 98
Thanks: 53
Thanked 126 Times in 57 Posts
Trích:
Nguyên văn bởi pexea12 View Post
Bài 7 Cho $a,b,c>0$ và $a+b+c=3$. Chứng minh rằng
$$\dfrac{1}{1+ab}+\dfrac{1}{1+bc}+\dfrac{1}{1+ca} \ge \dfrac{9}{2(\sqrt{a}+\sqrt{b}+\sqrt{c})}$$
TA có:
$$\frac{1}{1+ab}=1-\frac{ab}{1+ab}\geq 1-\frac{\sqrt{ab}}{2}$$
$$\frac{1}{1+ab}+\frac{1}{1+bc}+\frac{1}{1+ac}\geq 3-\frac{\sqrt{ab}+\sqrt{ac}+\sqrt{bc}}{2}=3-\frac{(\sqrt{a}+\sqrt{b}+\sqrt{c})^2-a-b-c}{4}$$
Ta cần chứng minh: $$\frac{15}{4}-\frac{(\sum \sqrt{a})^2}{4}\geq \frac{9}{2(\sum \sqrt{a})}$$
Đặt $t=\sqrt{a}+\sqrt{b}+\sqrt{c}$
Để ý rằng $$\sqrt{a+b+c}\leq \sqrt{a}+\sqrt{b}+\sqrt{c}\leq \sqrt{3(a+b+c)}$$
Nên $\Rightarrow \sqrt{3}<t\leq 3$ bất đẳng thức được viết lại thành
$$\frac{15}{4}-\frac{t^2}{4}\geq \frac{9}{2t}\Leftrightarrow (t-3)(t-\frac{\sqrt{33}-\sqrt{3}}{2})(t+\frac{\sqrt{33}-\sqrt{3}}{2})\leq 0$$
Dễ thấy $\sqrt{3}>\frac{\sqrt{33}-\sqrt{3}}{2}$ nên đánh giá đúng với $t \in (\sqrt{3};3]$
Phép chứng minh hoàn tất $\blacksquare$
------------------------------
Bài 8: Cho $a,b$ là các số thực dương thỏa $a+b=2$. Chứng minh rằng: $$\large {a^{a+ab}b^{b+ab}\geq 1}$$
[RIGHT][I][B]Nguồn: MathScope.ORG[/B][/I][/RIGHT]
 
__________________
$F\begin{Bmatrix}
\heartsuit
\end{Bmatrix}=\frac{1}{\sqrt{2\pi}}\int_{-\infty }^{+\infty }f(t)e^{it\heartsuit}dt=? $

thay đổi nội dung bởi: Ispectorgadget, 19-07-2012 lúc 06:06 PM Lý do: Tự động gộp bài
Ispectorgadget is offline   Trả Lời Với Trích Dẫn
The Following 3 Users Say Thank You to Ispectorgadget For This Useful Post:
greg_51 (13-06-2014), Hoang Nguyen (20-07-2012), lexuanthang (22-07-2012)
Old 19-07-2012, 06:23 PM   #15
pth_tdn
+Thành Viên+
 
Tham gia ngày: Dec 2009
Đến từ: HCM City
Bài gởi: 183
Thanks: 25
Thanked 240 Times in 122 Posts
Trích:
Nguyên văn bởi Ispectorgadget View Post
------------------------------
Bài 8: Cho $a,b$ là các số thực dương thỏa $a+b=2$. Chứng minh rằng: $$\large {a^{a+ab}b^{b+ab}\geq 1}$$
Ta cần chứng minh bất đẳng thức tương đương: $lna.(a+ab)+lnb(b+ba) \geq 0 \Leftrightarrow lna.a.(3-a)+lnb.b.(3-b) \geq 0 $.
Không mất tính tổng quát, giả sử $a \geq 1 $.
Ta có $b=2-a \leq \frac{1}{a} \Rightarrow ln(2-a) \leq -lna. $.
$\Rightarrow lna.a.(3-a)+lnb.b.(3-b) \geq lna.b.(3-b)+lna.a.(3-a)=lna.(2a-2) \geq 0 $, đpcm.
[RIGHT][I][B]Nguồn: MathScope.ORG[/B][/I][/RIGHT]
 
pth_tdn is offline   Trả Lời Với Trích Dẫn
The Following User Says Thank You to pth_tdn For This Useful Post:
hoang_kkk (20-07-2012)
Trả lời Gởi Ðề Tài Mới

Bookmarks

Ðiều Chỉnh
Xếp Bài

Quuyền Hạn Của Bạn
You may not post new threads
You may not post replies
You may not post attachments
You may not edit your posts

BB code is Mở
Smilies đang Mở
[IMG] đang Mở
HTML đang Tắt

Chuyển đến


Múi giờ GMT. Hiện tại là 11:56 AM.


Powered by: vBulletin Copyright ©2000-2024, Jelsoft Enterprises Ltd.
Inactive Reminders By mathscope.org
[page compression: 113.02 k/130.40 k (13.32%)]